Confusion about the derivation of acceleration relative to rotating frames

In summary: That's all it means.In summary, Kleppner's mechanics book explains how to find the acceleration in rotating coordinates by differentiating the equation ##\vec v_{in}=\vec v_{rot}+\vec\Omega\times\vec r##, where the subscripts IN and ROT refer to the inertial and rotating frames, respectively. The term ##\left(\frac{d\vec v_{in}}{dt}\right)_{rot}## is a way of expressing the velocity in the rotating frame, and is not equivalent to ##\frac{d\vec v_{
  • #1
Leo Liu
353
156
1610331742496.png

This derivation is found in Kleppner's mechanics book. It shows how to find the acceleration in rotating coordinates by differentiating ##\vec v_{in}=\vec v_{rot}+\vec\Omega\times\vec r##; subscripts IN and ROT stand for inertial and rotation respectively.
My question is what the term ##\left(\frac{d\vec v_{in}}{dt}\right)_{rot}## means. Is it the same thing as ##\frac{d\vec v_{rot}}{dt}##? Thank you.

Edit 1: It appears that someone has asked a similar question 12 years ago. So the question is now closed except for better explanations. Thanks!
https://www.physicsforums.com/threads/deriving-acceleration-in-rotating-reference-frames.286812/
Edit 2: Meh, it was of no help ¯\_(ツ)_/¯ .
 
Last edited:
  • Like
Likes Delta2
Physics news on Phys.org
  • #2
In Inertial frame it is interpreted as the quantity
[tex](\frac{dv_{in}}{dt})_{in}-\Omega \times v_{in}[/tex]
In rotationg frame it is interpreted as the quantity
[tex](\frac{dv_{rot}}{dt})_{rot}+\Omega \times v_{rot}[/tex]
With the second term in RHS, it is not genuine simple quantity neither in inertial frame or in rotating frame.
 
Last edited:
  • Like
Likes etotheipi
  • #3
Let ##xyz## be an inertial frame and let ##\xi\eta\zeta## stand for a rotating frame. Take any vector ##\boldsymbol u(t)## It follows that
$$\boldsymbol u(t)=u_x(t)\boldsymbol e_x+ u_y(t)\boldsymbol e_y+u_z(t)\boldsymbol e_z=U_\xi(t)\boldsymbol e_\xi+ U_\eta(t)\boldsymbol e_\eta+U_\zeta(t)\boldsymbol e_\zeta$$
Definition.
$$\frac{d}{dt}\Big|_{in}\boldsymbol u=\dot u_x(t)\boldsymbol e_x+ \dot u_y(t)\boldsymbol e_y+\dot u_z(t)\boldsymbol e_z$$
$$\frac{d}{dt}\Big|_{rot}\boldsymbol u=\dot U_\xi(t)\boldsymbol e_\xi+\dot U_\eta(t)\boldsymbol e_\eta+\dot U_\zeta(t)\boldsymbol e_\zeta$$
Theorem. Let ##\boldsymbol\omega## stand for the angular velocity of ##\xi\eta\zeta## relative xyz.
Then
$$\frac{d}{dt}\Big|_{in}\boldsymbol u=\frac{d}{dt}\Big|_{rot}\boldsymbol u+\boldsymbol\omega\times\boldsymbol u.$$
 
Last edited:
  • Like
  • Informative
Likes vanhees71, Leo Liu and etotheipi
  • #4
Thanks for your help! Could I ask a few questions?
wrobel said:
$$\boldsymbol u(t)=u_x(t)\boldsymbol e_x+ u_y(t)\boldsymbol e_y+u_z(t)\boldsymbol e_z=U_\xi(t)\boldsymbol e_\xi+ U_\eta(t)\boldsymbol e_\eta+U_\zeta(t)\boldsymbol e_\zeta$$
Does this equation represent the position vectors of a particle in two different frames and relate them with each other?
Is this equation the same as ##\vec r=\vec r',\, d\vec r\neq d\vec r'##?
 
  • #5
In that equation, ##\mathbf{u}## is any arbitrary vector, just written in terms of two sets of basis vectors which are rotating w.r.t. each other
 
  • Like
Likes Leo Liu
  • #6
Let we have a frame Oxyz and a frame Aξηζ. The position vector of the point M relative the first frame is OM and for the second frame is AM. It is clear OM=OA+AM. Correspondingly,
ddt|OxyzOM=ddt|OxyzOA+ddt|OxyzAM.
You can also write
ddt|OxyzAM=ddt|AξηζAM+ω×AM
Here
ddt|AξηζAM is the velocity of M relative Aξηζ
 
  • Like
Likes Leo Liu
  • #7
I will not retype it. This forum converts text spontaneously
 
  • #8
It's a known bug. I fixed it up for you :smile:
wrobel said:
Let we have a frame ##Oxyz## and a frame ##A \xi \eta \zeta## . The position vector of the point M relative the first frame is ##\boldsymbol{OM}## and for the second frame is ##\boldsymbol{AM}##. It is clear ##\boldsymbol{OM}=\boldsymbol{OA}+\boldsymbol{AM}##. Correspondingly,$$\frac{d}{dt} \Big|_{Oxyz} \boldsymbol{OM}= \frac{d}{dt}\Big|_{Oxyz} \boldsymbol{OA}+\frac{d}{dt}\Big|_{Oxyz} \boldsymbol{AM}$$You can also write$$ \frac{d}{dt}\Big|_{Oxyz} \boldsymbol{AM}=\frac{d}{dt}\Big|_{A \xi \eta \zeta} \boldsymbol{AM} + \boldsymbol{\omega} \times \boldsymbol{AM}$$
Here ##\frac{d}{dt}\Big|_{A \xi \eta \zeta} \boldsymbol{AM}## is the velocity of M relative ##A \xi \eta \zeta##
 
  • Like
  • Love
Likes Nugatory, wrobel, Leo Liu and 1 other person
  • #9
wrobel said:
I will not retype it. This forum converts text spontaneously
It's a known bug. I think keeping the editor in plain text mode (click the cogwheel at the right hand end of the editor toolbar so all other icons are disabled) helps, since then the forum doesn't try to translate BBCODE/LaTeX in the editor.
 
  • Like
Likes wrobel and etotheipi
  • #10
(Double edit: I've mightily scuffed the chronological ordering of this thread, but I'm going to leave this reply up because I don't want to delete any of my sexy ##\LaTeX## 😅)

Let's say we have the two bases, the body-fixed ##\{ \mathbf{e}_a \}## and the space-fixed ##\{ \tilde{\mathbf{e}}_a \}##, as per Tong's notation. Any vector ##\mathbf{u}##, you can express as$$\mathbf{u} = u^a \mathbf{e}_a = \tilde{u}^a \tilde{\mathbf{e}}_a$$We can take the derivative, treating the space-fixed ##\tilde{\mathbf{e}}_a## as constant, i.e.$$ \left( \frac{d\mathbf{u}}{dt} \right)_{\text{space}} = \frac{du^a}{dt} \mathbf{e}_a + u^a \frac{d\mathbf{e}_a}{dt} = \frac{d\tilde{u}^a}{dt} \tilde{\mathbf{e}}_a$$Tong used the example where the body is undergoing rotation about a fixed point which is the origin of both frames, and where the vector ##\mathbf{u} := \mathbf{r} ## is the position vector, in which case the space-fixed coordinates are constant ##\frac{d r^a}{dt} = 0##. That's all perfectly fine.

Kleppner is going one step further here, and thinking about taking the time derivative, treating the body-fixed ##\mathbf{e}_a## as constant, i.e.$$ \left( \frac{d\mathbf{u}}{dt} \right)_{\text{body}} = \frac{du^a}{dt} \mathbf{e}_a = \frac{d\tilde{u}^a}{dt} \tilde{\mathbf{e}}_a + \tilde{u}^a \frac{d\tilde{\mathbf{e}}_a}{dt}$$That's all the bracket notation means, it's telling you which basis vectors you're treating as constant. We can combine these two equations,$$\left( \frac{d\mathbf{u}}{dt} \right)_{\text{space}} = \left( \frac{d\mathbf{u}}{dt} \right)_{\text{body}} + u^a \frac{d\mathbf{e}_a}{dt}$$but notice that since we can express$$\frac{d\mathbf{e}_a}{dt}= \boldsymbol{\omega} \times \mathbf{e}_a \implies u^a \frac{d\mathbf{e}_a}{dt} = \boldsymbol{\omega} \times u^a \mathbf{e}_a = \boldsymbol{\omega} \times \mathbf{u}$$we have$$\left( \frac{d\mathbf{u}}{dt} \right)_{\text{space}} = \left( \frac{d\mathbf{u}}{dt} \right)_{\text{body}} + \boldsymbol{\omega} \times \mathbf{u}$$
 
Last edited by a moderator:
  • Like
Likes Leo Liu
  • #11
Update: I finally understand the logic behind this derivation after reading Tong's notes. I coundln't grasp the idea because the notation ##(d/dt)_{in/rot}## was novel and confusing to me. Tong dispeled my confusing by sticking with the old-school notation. This is what I leant:
We first describe the position of a particle in two frames.
Inertial:
$$\vec r=r_1\hat e_1+r_2\hat e_2+r_3\hat e_3$$
Rotating:
$$\vec r=r_1'\hat e_1'+r_2'\hat e_2'+r_3'\hat e_3'$$
To find the velocity, we have to differentiate them. The first one is quite staightforward, while we need to apply chain rule to the second one because the base vectors of a rotating frame could be moving.
Therefore,
$$\begin{aligned}
\dot{\vec r}=\underbrace{\sum_{i=1}^3r_i\hat e_i}_\text{v in stationary frame}&=\underbrace{(\dot r_1',\dot r_2',\dot r_3') \langle \hat e_1',\hat e_2',\hat e_3'\rangle+(r_1',r_2',r_3') \langle \dot{\hat e_1'},\dot{\hat e_2'},\dot{\hat e_3'}\rangle}_\text{v in rotating frame}\\
&=\sum_{i=1}^3 \dot r_i'\hat e_i'+r_i\cdot\underbrace{\langle \omega_1,\omega_2,\omega_3\rangle\times\langle \hat e_1',\hat e_2',\hat e_3'\rangle}_{\because\,\vec v=\vec\omega\times\vec r}
\end{aligned}$$
This is the true velocity in two frames.
We have to differentiate it again to find out the acceleration.
$$\ddot{\vec r}=\frac{d\sum_{i=1}^3 \dot r_i'\hat e_i'+\omega_i\times r_i\hat e_i'}{dt}$$
$$
\begin{aligned}
a_{in}=&\sum_{i=1}^3 \ddot{r_i'}\hat e_i'+\underbrace{\dot r_i'\dot{\hat e_i'}}_{=\vec\omega_i\times\dot r_i'\dot{\hat e_i'}}+\underbrace{\underbrace{\dot{\vec\omega}\times r_i'\hat e_i'}_{=0\text{ if frame rotates at constant speed}}+\vec\omega\times r_i'\dot{\vec e_i'}+\vec\omega\times\dot r_i'\hat e_i'}_\text{3 terms chain rule}\\
=&\vec a_{rot}+2\vec\omega\times\vec v_{rot}+\vec\omega\times(\vec\omega\times\vec r)
\end{aligned}$$
This is the equation that relates acc in rotating frame and acc in inertial frame; the condition is that the frame rotates at a constant rate.
I hope this post can help someone who may have the same question in the future.
 
  • Like
Likes etotheipi
  • #12
etotheipi said:
(Edit: this is a reply to a post that was deleted, but hopefully it will still be useful 😅)

@Leo Liu it's worth noting that what Tong is doing is slightly different to what Kleppner is doing. Let's say we have the two bases, the body-fixed ##\{ \mathbf{e}_a \}## and the space-fixed ##\{ \tilde{\mathbf{e}}_a \}##, as per Tong's notation. Any vector ##\mathbf{u}##, you can express as$$\mathbf{u} = u^a \mathbf{e}_a = \tilde{u}^a \tilde{\mathbf{e}}_a$$We can take the derivative, treating the space-fixed ##\tilde{\mathbf{e}}_a## as constant, i.e.$$ \left( \frac{d\mathbf{u}}{dt} \right)_{\text{space}} = \frac{du^a}{dt} \mathbf{e}_a + u^a \frac{d\mathbf{e}_a}{dt} = \frac{d\tilde{u}^a}{dt} \tilde{\mathbf{e}}_a$$Tong used the example where the body is undergoing rotation about a fixed point which is the origin of both frames, and where the vector ##\mathbf{u} := \mathbf{r} ## is the position vector, in which case the space-fixed coordinates are constant ##\frac{d r^a}{dt} = 0##. That's all perfectly fine.
Meh, how could you type latex equations that fast :woot:?
 
  • Haha
Likes etotheipi
  • #13
The confusion is just to make it more complicated than it is. You have an inertial reference frame with a (by definition) time-independent cartesian basis ##\vec{e}_j## and a cartesian basis fixed in the rotating frame, which is time dependen, ##\vec{e}_k'(t)##. Of course there's a time-dependent rotation matrix ##D_{jk}(t)## connecting the two bases (here and in the following Einstein summation convention applies):
$$\vec{e}_k'(t)=\vec{e}_j D_{jk}(t).$$
Now we have
$$\dot{\vec{e}}_k'=\vec{e}_j \dot{D}_{jk} = \vec{e}_l' D_{jl} \dot{D}_{jk}.$$
Further
$$D_{jl} D_{jk}=\delta_{jk},$$
because ##(D_{jk}) \in \mathrm{SO}(3)##. and thus
$$\Omega_{lk}'=D_{jl} \dot{D}_{jk}=-\Omega_{kl}'=\epsilon_{klm} \omega_m'=\epsilon_{lmk} \omega_m'.$$
So for the derivative of a vector you get
$$\dot{\vec{V}}=\mathrm{d}_t (V_j' \vec{e}_j')=\dot{V}_j' \vec{e}_j' + V_j' \dot{\vec{e}}_j' = \dot{V}_j' \vec{e}_j' + V_j' D_{kj} \dot{D}_{kl} \vec{e}_l'=(\dot{V}_l' + \epsilon_{lmj} \omega_m' V_j')\vec{e}_l'.$$
Now you define a "covariant time derivative" for vector components wrt. the rotating basis by
$$\dot{\vec{V}}=\vec{e}_l' \mathrm{D}_t V_l',$$
leading to
$$\mathrm{D}_t V_l' = \dot{V}_l' + (\vec{\omega}' \times \vec{V}')_l.$$
 
  • Like
Likes etotheipi

What is a rotating frame?

A rotating frame is a coordinate system that is rotating with respect to an inertial frame of reference. This means that the origin and axes of the rotating frame are moving and changing over time.

How is acceleration defined in a rotating frame?

In a rotating frame, acceleration is defined as the rate of change of velocity with respect to time, measured from the perspective of the rotating frame. This means that an object may appear to be accelerating in a rotating frame, even if it is not experiencing any external forces.

Why is there confusion about the derivation of acceleration in rotating frames?

There is confusion about the derivation of acceleration in rotating frames because it involves the use of non-inertial reference frames, which can be counterintuitive and difficult to understand. Additionally, the equations used to calculate acceleration in rotating frames may differ from those used in inertial frames, leading to further confusion.

How can one calculate acceleration in a rotating frame?

To calculate acceleration in a rotating frame, one must first identify the rotating frame and determine the angular velocity of the frame. Then, the equations of motion can be used to calculate the acceleration of an object in the frame.

What are some real-life examples of acceleration in rotating frames?

Some examples of acceleration in rotating frames include the centrifugal force experienced by objects in a rotating amusement park ride, the Coriolis force acting on objects on a rotating Earth, and the apparent acceleration of objects on a rotating carousel.

Similar threads

Replies
2
Views
673
Replies
30
Views
2K
Replies
4
Views
745
Replies
7
Views
282
Replies
17
Views
1K
Replies
2
Views
733
Replies
2
Views
1K
Replies
25
Views
1K
Back
Top